Why Does E(X1 + X2 + ... + Xn) Equal n*E(Xi) in Statistics?

  • Thread starter Thread starter cdux
  • Start date Start date
  • Tags Tags
    Statistics
Click For Summary
The discussion revolves around proving that the expected value of the unbiased estimator Theta_n equals Theta for a uniform distribution U(0, Theta). It clarifies that E(X1 + X2 + ... + Xn) equals n * E(Xi) because all Xi are identically distributed, leading to the result that E(Theta_n) simplifies to Theta. Additionally, the conversation touches on the variance of the sum of uncorrelated variables, establishing that Var(X1 + X2 + ... + Xn) equals nVar(Xi) due to the property of variance for uncorrelated random variables. The participant initially struggled with these concepts but ultimately found clarity in the properties of expected value and variance. Understanding these statistical properties is crucial for accurate calculations in statistics.
cdux
Messages
187
Reaction score
0
I'm a bit confused about a particular step in a calculation.

Given Theta_n = (2/n)(X1 + X2 + ... + Xn) being an unbiased estimator of Theta for a U(0,Theta), we have to prove it by showing E(Theta_n) = Theta.

And we go on E(Theta_n) = (2/n)E(X1+X2 + .. Xn)

Now at this point the solution is (2/n) * n * (Theta/2) (= Theta which is the sought-after result)

I understand that Theta/2 is the mean of a U() but how exactly does one go from E(X1 + X2 + .. Xn) to equaling it to n*E(Xi)? Is E(X1) = E(X2) = E(Xi)? If yes, why?

(PS. A more complex example is Var(X1+X2 + .. Xn) appearing to also result to nV(Xi) (=nσ^2) )
 
Physics news on Phys.org
Hrm. Afterthought. I guess it might be simply a blatant case of "mean of the whole but it missed the 'over n' so it's n * E(Xi)".

I guess it might apply in the case of Var too..
 
I'm having a difficulty seeing how that could be true for Var.

Var((2/n)(X1 + X2 + ... + Xn)) = (4/n^2)nVar(Xi).

I understand 4/n^2 going out as a property of Var but how is Var(X1 + X2 + ... + Xn) = nVar(Xi)?
 
Nevermind. I found it. If X1, X2.. are uncorrelated then V(Σ(Xi)) = ΣV(Xi) ..after a proof involving Var's equality with E[X^2) - E^2
 
Question: A clock's minute hand has length 4 and its hour hand has length 3. What is the distance between the tips at the moment when it is increasing most rapidly?(Putnam Exam Question) Answer: Making assumption that both the hands moves at constant angular velocities, the answer is ## \sqrt{7} .## But don't you think this assumption is somewhat doubtful and wrong?

Similar threads

Replies
9
Views
3K
  • · Replies 7 ·
Replies
7
Views
18K
  • · Replies 5 ·
Replies
5
Views
3K
Replies
1
Views
3K
  • · Replies 1 ·
Replies
1
Views
2K
Replies
4
Views
2K
  • · Replies 1 ·
Replies
1
Views
2K
  • · Replies 3 ·
Replies
3
Views
2K
Replies
3
Views
2K
  • · Replies 2 ·
Replies
2
Views
2K